PDF thống nhất về sự khác biệt của hai rv


9

Có thể có bản PDF về sự khác biệt của hai iid rv giống như một hình chữ nhật (thay vì, hình tam giác chúng ta có được nếu các rv được lấy từ phân phối đồng đều).

tức là có thể PDF f của jk (đối với hai iid rv được lấy từ một số phân phối) có f (x) = 0,5 cho tất cả -1 <x <1?

Không có giới hạn nào đối với phân phối mà chúng tôi lấy j và k từ đó ngoại trừ min là -1 và max là 1.

Sau một vài thử nghiệm, tôi nghĩ điều này có thể là không thể.


Sự khác biệt của hai phân phối đồng phục là phân phối tam giác, vì vậy nếu bạn hỏi liệu có thể nhận được đồng phục về sự khác biệt của đồng phục iid không, thì câu trả lời là không.
Tim

Cùng Q hỏi ở đây: math.stackexchange.com/questions/2036139/ cho đến nay mà không có câu trả lời!
kjetil b halvorsen

Thực sự có vẻ khó tránh khỏi việc nhận ra bên ngoài khi cả và có khối lượng xác suất gần với các điểm cuối này. j k[1,1]jk
Christoph Hanck

2
Điều đó là không thể. Theo hồi ức của tôi, điều này là (ở dạng hơi khác) đã được trả lời ở đâu đó trên trang web. Tôi sẽ xem liệu tôi có thể xác định vị trí của nó không
Glen_b -Reinstate Monica

1
@Glen_b Bạn có thể đang nhớ lại thống kê.stackexchange.com/questions/125360/ . Tuy nhiên, đây không hoàn toàn là một bản sao vì sự khác biệt của các biến iid, mặc dù có thể biểu thị dưới dạng tổng có thể liên quan đến tổng các biến có phân phối không giống nhau. Tôi tin rằng một sửa đổi nhỏ trong giải pháp của tôi sẽ giải quyết sự khác biệt này; Giải pháp của Silverfish có vẻ như được áp dụng trực tiếp mà hầu như không sửa đổi, nhưng trước tiên người ta phải loại bỏ rất nhiều vật liệu bên ngoài để thấy điều đó. X + ( - Y ) ,XYX+(Y),
whuber

Câu trả lời:


10

Định lý: Không có phân phối mà khi .DistABU(1,1)A,BIID Dist


Chứng minh: Xét hai biến ngẫu nhiên với hàm đặc trưng chung . Biểu thị sự khác biệt của chúng bằng . Chức năng đặc trưng của sự khác biệt là:A,BIID DistφD=AB

φD(t)=E(exp(itD))=E(exp(it(AB)))=E(exp(itA))E(exp(itB))=φ(t)φ(t)=φ(t)φ(t)¯=|φ(t)|2.

(Dòng thứ tư của công việc này xuất phát từ thực tế là hàm đặc trưng là Hermiti .) Bây giờ, lấy đưa ra một hình thức cụ thể cho , đó là:DU(1,1)φD

φD(t)=E(exp(itD))=Rexp(itr)fD(r)dr=1211exp(itr)dr=12[exp(itr)it]r=1r=1=12exp(it)exp(it)it=12(cos(t)+isin(t))(cos(t)+isin(t))it=12(cos(t)+isin(t))(cos(t)isin(t))it=122isin(t)it=sin(t)t=sinc(t).

trong đó cái sau là hàm chân (không chuẩn hóa ) . Do đó, để đáp ứng các yêu cầu cho , chúng tôi yêu cầu một hàm đặc trưng với định mức bình phương được đưa ra bởi:Distφ

|φ(t)|2=φD(t)=sinc(t).

Phía bên trái của phương trình này là một chỉ tiêu bình phương và do đó không âm, trong khi phía bên phải là một hàm âm ở nhiều nơi khác nhau. Do đó, không có giải pháp cho phương trình này, và do đó không có hàm đặc trưng thỏa mãn các yêu cầu cho phân phối. (Hat-tip to Fabian vì đã chỉ ra điều này trong một câu hỏi liên quan đến Toán học .) Do đó, không có phân phối với các yêu cầu của định lý.


3

Đây là một kỹ sư điện đảm nhận vấn đề này, với quan điểm phù hợp hơn với dsp.SE hơn là chỉ số.SE, nhưng không vấn đề gì.

Giả sử và là các biến ngẫu nhiên liên tục với pdf phổ biến . Sau đó, nếu biểu thị , chúng ta có Bất đẳng thức Cauchy - Schwarz cho chúng ta biết rằng có cực đại tại . Trong thực tế, vì thực sự là hàm "tự tương quan" của được coi là "tín hiệu", nên nó phải có một mức tối đa duy nhất tại và do đó không thể được phân phối đồng đều như mong muốn. Ngoài ra, nếuXYf(x)ZXY

fZ(z)=f(x)f(x+z) dx.
fZ(z)z=0fZfz=0Z fZthực sự là mật độ đồng nhất (hãy nhớ rằng đó cũng là hàm tự tương quan), thì "mật độ phổ công suất" của (được coi là tín hiệu) sẽ là một hàm chân, và do đó không phải là hàm không âm vì tất cả các mật độ phổ công suất phải là . Ergo, giả định rằng là mật độ đồng đều dẫn đến mâu thuẫn và vì vậy giả định phải sai.fZfZ

Khiếu nại rằng rõ ràng không hợp lệ khi phân phối chung của và chứa các nguyên tử vì trong trường hợp như vậy, phân phối cũng sẽ chứa các nguyên tử. Tôi nghi ngờ rằng hạn chế và có pdf có thể được gỡ bỏ và bằng chứng lý thuyết đo lường hoàn toàn được xây dựng cho trường hợp chung khi và không nhất thiết phải thưởng thức pdf (nhưng sự khác biệt của chúng là).X Y Z X Y X YfZU[1,1]XYZXYXY


1
Một phần trong đó dường như không đúng với tôi. Hàm đặc trưng của phân phối hàm , vì vậy rõ ràng loại biến đổi Fourier được cho phép. Theo tôi, logic của bạn dẫn đến việc chứng minh quá nhiều - dường như không chỉ chứng minh rằng không thể đồng nhất mà cả phân phối đồng đều hoàn toàn không tồn tại. Có phải tôi đã hiểu lầm? sinc ZU(1,1)sincZ
Ben - Tái lập Monica

1
Có hay không chức năng đặc trưng của không phải là vấn đề; nó tồn tại Pdf của là một hàm tự tương quan . Vâng, mật độ phổ công suất của bất kỳ chức năng tự tương quan phải là một hàm không âm. Vì vậy, giả định rằng dẫn đến mật độ phổ công suất là hàm chân (đảm nhận cả hai giá trị dương và âm). Vì đây không phải là mật độ phổ công suất hợp lệ (hãy nhớ rằng cũng là một hàm tự tương quan), giả định rằng phải sai.Z f ZU [ - 1 , 1 ] f Z f ZU [ - 1 , 1 ]U[1,1]ZfZU[1,1]fZfZU[1,1]
Dilip Sarwate
Khi sử dụng trang web của chúng tôi, bạn xác nhận rằng bạn đã đọc và hiểu Chính sách cookieChính sách bảo mật của chúng tôi.
Licensed under cc by-sa 3.0 with attribution required.